, peds nclex

Ace your homework & exams now with Quizwiz!

In discussing the treatment for children with scoliosis, a group of pediatric nurses makes the following statements. Which statement is most accurate related to the treatment of scoliosis? a) "The only successful treatment for scoliosis is surgery within two weeks of the diagnosis." b) "Children treated for scoliosis by using braces have to wear the brace almost all the time." c) "Children with severe scoliosis are treated using electrical stimulation." d) "The treatment for children with scoliosis usually lasts three to four months."

"Children treated for scoliosis by using braces have to wear the brace almost all the time." Correct Explanation: The Boston brace and the TLSO brace are made of plastic and are customized to fit the child for treatment of scoliosis. The brace should be worn constantly, except during bathing or swimming, to achieve the greatest benefit.

A nurse is assisting the parents of a child who requires a Pavlik harness. The parents are apprehensive about how to care for their baby and concerned about holding and playing with him. How can the nurse best assist the parents? a) "The baby only needs the harness for 12 weeks." b) "Do not attempt to adjust the harness yourself." c) "Let's put you in touch with other families who have experienced this." d) "The harness does not hurt the baby."

"Let's put you in touch with other families who have experienced this." Correct Explanation: There are many helpful pointers and suggestions that are available from other parents and orthopedic organizations. Referring the parents to other families who have experienced a Pavlik harness will provide assurance and likely increase compliance with the regimen. The other responses are factual but do not address the parent's concerns.

The caregiver of a 2-year-old who has a polyurethane resin cast on her arm calls the clinic to report that her child is crying and says that the cast has sand in it. The caregiver states that she has had casts of her own and knows how badly they can itch. She says she always used a hanger to scratch but is worried that it will be too sharp for the child. Which of the following statements would be appropriate for the nurse to make to this caregiver? a) "Since the child's cast is synthetic, she could soak it with cool water." b) "A plastic ruler is less likely than a hanger to cut the child's skin." c) "You could give the child an extra dose of acetaminophen and see if that helps." d) "Nothing should be put into the cast. You can blow cool air into it with a hair dryer."

"Nothing should be put into the cast. You can blow cool air into it with a hair dryer." Correct Explanation: Children and caregivers should be cautioned not to put anything inside the cast, no matter how much the casted area itches. Small toys and sticks or stick-like objects should be kept out of reach until the cast has been removed. Ice packs applied over the cast may help decrease the itching. Blowing cool air through a cast with a hair dryer set on a cool temperature or using a fan may help to relieve discomfort under a cast.

The nurse has provided teaching to a family regarding clean intermittent catheterization for their 4 year-old daughter with myelomeningocele. Which statement, if made by the family, indicates that further teaching is required by the nurse? Select all that apply: 1) "When the catheter becomes stiff, we will soak it in warm water to soften it." 2) "We will store our catheter in a zip-top plastic bag." 3) "We will insert the catheter about 4-6 inches, or until we see urine flow." 4) "We will lubricate the catheter with a petroleum-based jelly before usage." 5) "We will clean our daughter's genitalia with a washcloth before catheterization."

1, 3, 4 When catheters become stiff, cloudy, rough, cracked, or damaged in general, they need to be thrown away. Catheters should be inserted approximately 2-3 inches for females and 4-6 inches for males, or until urine flow is noted. A water-based lubricator needs to be used. See page 809.

To prevent rupture of the sac in a patient born with myelomeningocele, which of the following treatments does the nurse anticipate? 1) Ultrasound 2) Surgical closure 3) Banding to form necrotic tissue 4) Insertion of drain to collect CSF leakage

2) Page 807.

A nurse admits a child who has a history cerebral palsy. Which assessment finding by the nurse is most concerning? 1) The mother reports the child had a seizure 5 hours ago. 2) The child has a fever of 100.3. 3) The child is standing on his toes. 4) The mother reports the child's twisting movements seem to have worsened since arriving at the clinic.

2) This fever could indicate aspiration pneumonia, and this needs to be investigated further immediately with questioning of coughing, respiratory difficulty, or sputum production. Seizures are common with cerebral palsy. The child's symptoms have likely worsened because of the stress due to a clinic visit. Standing on the toes or scooting on the back (instead of crawling on the abdomen) are both commonly seen in a patient with cerebral palsy.

The nurse is caring for a patient diagnosed with a meningocele. The nurse should perform all of the following actions except: 1) Documenting the presence of a sac protruding from the lower spinal column. 2) Documenting the presence of clear fluid draining from the meningocele. 3) Encouraging fluids hourly. 4) Measuring head circumference every shift.

2) This may indicate a CSF leak and should be reported

When assessing a child for developmental dysplasia of the hip, the nurse feels a "clunk" when the child's hip is abducted and relocated. Which of the following did the nurse perform? 1) Braxton's maneuver 2) Ortolani's sign 3) Trendelenburg's sign 4) Barlow's test

2) Barlow's is when the hip is adducted and dislocated.

A nurse has just finished providing discharge teaching to the family of a child going home with a cast that was applied 30 minutes prior. Which statement by the family indicates that further teaching is necessary? Select all that apply: 1) "For the next couple of days, we should keep the casted arm above the level of the heart and apply ice." 2) "If my child's arm starts to itch, we can apply lotion if we can reach into the cast." 3) "We will tape a plastic bag around his cast before he takes a bath." 4) "If my child gets his cast wet, we'll blow dry it with the lowest heat setting on our blow dryer." 5) "We will make sure we regularly press the skin back around the edge of the cast." 6) "We will make sure our child does not eat anything messy while he has this cast on."

2, 4, 6 Nothing should be inserted into the cast, and lotions and powders should not be used. If the cast gets wet, a blow dryer with COLD air should be used. There is no need to adjust the child's diet, but the cast should be covered while the child eats or drinks. See page 844

While caring for a 9-year-old female in Buck's traction, which of the following actions by the nurse is correct? 1) The nurse encourages the child's 3 year-old sibling to sit on the bed and visit with the child. 2) The nurse helps the child learn how to raise and lower the head of her bed so she can complete her homework. 3) The nurse checks the capillary refill on the child's extremities every 4 hours. 4) The nurse teaches the child's mother to place the weights on the bedside table before the child uses the bedpan.

3) Extra visitors should not be invited on the bed- especially a toddler who may think the weights at the end of the bed are toys. The head of the bed should only be raised or lowered with physician's orders, and this should be done minimally. The weights should ALWAYS be hanging freely.

Which diagnostic exam does the nurse know will best aid in the diagnosis of Duchenne muscular dystrophy? 1) EEG 2) CT Scan 3) MRI 4) EMG

4)

The day shift pediatric nurse receives report on the following 4 patients. After receiving report, which patient should the nurse assess first? 1) The 4-year-old female with cerebral palsy who admits with difficulty swallowing and a fever of 101.9. 2) The 9-month-old male with myelomeningocele who has absent deep tendon reflexes in bilateral lower extremities. 3) The 6-year-old male with Duchenne muscular dystrophy who appears sad and withdrawn. 4) The 7-year-old female with a spinal cord injury who reports numbness and tingling in her feet.

4) Spinal cord injury is a medical emergency that requires immediate assessment. A fever and difficulty swallowing in a child with cerebral palsy may indicate aspiration pneumonia, so this child should be seen next. Absent deep tendon reflexes are expected in a child with myelomeningocele, related to the paralysis seen below the sac. A child with Duchenne muscular dystrophy may be sad or withdrawn due to corticosteroid medication side effects or chronicity of the disease.

A concerned mother calls the and tells you the following pieces of information about her 2-year-old son. Which statement by the mother most concerns you? 1) "I noticed that when my son is standing, his knees touch but his feet seem really far apart." 2) "My son's feet are so flat, even though he's been waking for 9 months now." 3) "My baby really hates it when I try to feed him broccoli and keeps spitting it out onto his plate." 4) "This morning when I was trying to dress him, my son cried nonstop when I tried to put his shirt on."

4) This could indicate a sprain or fracture, and nursemaid's elbow is common in toddlers and preschoolers. This statement should be further investigated since it indicates pain/discomfort. It is not uncommon for infants to have flat feet, although the arch of the foot should begin to form after walking begins. However, some infants never develop an arch and have flat feet as adults. Genu valgum, or "knock knees", are commonly seen around the ages 2-3, and this will often resolve by ages 7-8. It is not uncommon for a toddler to dislike broccoli. Who does like broccoli.

The nurse is assisting the health care provider (HCP) examining an infant with developmental dysplasia of the hip perform an Ortolani maneuver. The nurse knows that this maneuver is performed for which purpose? A. To assess for hip instability B. To assess for movement of the hips C. To push the femoral head into the acetabulum D. To ensure full range of motion exists

A

The nurse teaching the parent of a child diagnosed with systemic lupus erythematosus (SLE). The nurse evaluates the teaching as effective when the parent states: A. "The cause is unknown." B. "There is no genetic involvement." C. "Drugs are not a trigger for the illness." D. "Antibiotics improve disease outcome."

A

Which will help the school-aged child with muscular dystrophy stay active longer? A. Normal activities, such as swimming B. Using a treadmill every day C. Several periods of rest every day D. Using a wheelchair upon getting tired

A

Why are chemotherapeutic agents such as methotrexate and cyclophosphamide sometimes used to treat JIA? A. Affect the immune system B. Effective against cancer-like JIA C. Are similar to NSAIDS D. Are absorbed into the synovial fluid

A

A mother whose 7-year-old child has been placed in a cast for a fractured right arm reports he will not stop crying even after taking Tylenol with codeine. He also will not straighten the fingers on his right arm. The nurse tells the mother to do which? a. Take him to the emergency department. b. Put ice on the injury. c. Avoid letting him get so tired. d. Wait another hour. If he is still crying, call back.

A Unrelieved pain and the childs inability to extend his fingers are signs of compartmental syndrome, which requires immediate attention. Placing ice on the extremity is an inappropriate action for the presenting symptoms. It is inappropriate for the nurse to tell the mother who is concerned about her child to avoid letting him get so tired. A child who has signs and symptoms of compartmental syndrome should be seen immediately. Waiting an hour could compromise the recovery of the child.

The nurse is caring for a group of children on the pediatric unit. The nurse should collect further data and explore the possibility of child abuse in which of the following situations? a) A 7-year-old with a spiral fracture of the humerus, which the caregiver reports as having been caused when the child was hit by a bat swung by a Little League teammate. b) A 9-year-old with a compound fracture of the tibia, which the caregiver reports as having been caused when the child attempted a flip on a skateboard. c) A 10-year-old with a simple fracture of the femur, which the caregiver reports as having been caused when the child fell down a set of stairs. d) A 6-year-old with a greenstick fracture of the wrist, which the caregiver reports as having been caused when the child fell while ice-skating.

A 7-year-old with a spiral fracture of the humerus, which the caregiver reports as having been caused when the child was hit by a bat swung by a Little League teammate. Correct Explanation: Spiral fractures, which twist around the bone, are fre quently associated with child abuse and are caused by a wrenching force. When a broken bone penetrates the skin, the fracture is called compound, or open. A simple, or closed, fracture is a single break in the bone without penetration of the skin. In a greenstick fracture, the bone bends and often just partially breaks.

What is the most accurate diagnostic tool in diagnosing suspected osteomyelitis in the pediatric patient? A. bone scan B. RBC count C. X-ray D. serum albumin level

A. bone scan

What are appropriate interventions when caring for a child in traction? (Select all that apply.) A. use of trapeze for positioning B. neurovascular checks performed regularly C. upright for 30 minutes a day D. skin integrity monitored regularly E. other extremities must be immobilized F. liquid diet to prevent constipation

A. use of trapeze for positioning B. neurovascular checks performed regularly D. skin integrity monitored regularly

A 5-year-old has recently been diagnosed with JIA. The nurse is reviewing his diagnostic findings from his blood work. Which of the following findings would the nurse expect to see? Select all that apply. A. Elevated sedimentation rate (ESR) B. Positive antinuclear antibody test (ANA) C. C-reactive protein (CRP) greater than 10mg/L D. Positive rheumatoid factor test E. Low vitamin D levels F. Low hemoglobin levels

ABCD

The nurse is working with a child who has just had a fracture reduction and casting. The nurse knows that the child is at risk for compartment syndrome. What objective assessment items will the nurse monitor for while completing the nursing assessment? Select all that apply. A. Burning B. Fever C. Weak pulse D. Tingling E. Pale grey extremity

ACDE

When planning a rehabilitative approach for a child with osteogenesis imperfecta (OI), the nurse should work to prevent which of the following? Select all that apply. A. Positional contractures and deformities B. Bone infection C. Muscle weakness D. Osteoporosis E. Misalignment of lower extremity joints

ACDE

Which factors are associated with slipped capital femoral epiphysis (SCFE)? Select all that apply. A. Obesity B. Female gender C. African descent D. Age of 5-10 years E. Pubertal hormonal changes F. Endocrine disorders

AEF

A group of students are reviewing information about the skeletal development in children. The students demonstrate understanding of the information when they identify that ossification is complete by what age? a) Preschool age b) School age c) Adolescence d) Toddlerhood

Adolescence Correct Explanation: Ossification and conversion of cartilage to bone continue throughout childhood and are complete at adolescence.

You assist with the application of a full-body plaster cast to a child. The child immediately becomes diaphoretic and complains of being hot. Which nursing intervention would be indicated? a) Moisten the cast with cool water. b) Observe the child for infection. c) Advise the child that this is to be expected. d) Suggest removal of the cast to the orthopedist.

Advise the child that this is to be expected. Correct Explanation: Plaster becomes hot as it sets. This effect is reduced with newer plastic casts. This is a normal expectation about which to educate the child before the application of the cast. If discomfort continues, notify the provider. Infection would not present in this way with a cast application. Never moisten a case.

The nurse is especially concerned to assess for adequate respiratory function in which of the following disease processes? Select all that apply: a) Spina bifida occulta b) Duchene muscular dystrophy c) Spinal Muscular Atrophy d) Brachial plexus injury e) Cerebral Palsy

Answer: B, C, and E.

A nurse applies ice to a patient's leg to relieve the pain due to a soft tissue injury. Which of the following is a recommended guideline for use of cold therapy? a) Apply gel packs for no longer than 30 minutes and monitor closely for tissue damage. b) Apply ice for the first 24 to 48 hours after injury to reduce edema.

Apply ice for the first 24 to 48 hours after injury to reduce edema. Explanation: Application of ice is recommended for the first 24 to 48 hours after injury to reduce edema. Ice is usually applied as crushed ice in a bag, applied over a thin layer of cloth, or by immersing the injured part in cool water. Cold therapy is not recommended for persons with hypersensitivity to cold or with impaired circulation. Gel packs cool skin faster than an ice bag and should be applied for no more than 10 minutes.

When assessing a child for an upper extremity fracture, the nurse should know that these fractures most often result from: a. automobile accidents. b. falls. c. physical abuse. d. sports injuries.

B The major cause of childrens fractures is falls. Because of the protection reflexes, the outstretched arm often receives the full force of the fall. Automobile accidents, physical abuse, and sports injuries may result in fractures to any bone.

When infants are seen for fractures, which nursing intervention is a priority? a. No intervention is necessary. It is not uncommon for infants to fracture bones. b. Assess the familys safety practices. Fractures in infants usually result from falls. c. Assess for child abuse. Fractures in infants are often nonaccidental. d. Assess for genetic factors.

C Fractures in infants warrant further investigation to rule out child abuse. Fractures in children younger than 1 year are not common because of the cartilaginous quality of the skeleton; a large amount of force is necessary to fracture their bones. Infants should be cared for in a safe environment and should not be falling. Fractures in infancy are usually nonaccidental rather than related to a genetic factor.

Which is an accurate statement concerning a childs musculoskeletal system and how it may be different from adults? a. Growth occurs in children as a result of an increase in the number of muscle fibers. b. Infants are at greater risk for fractures because their epiphyseal plates are not fused. c. Because soft tissues are resilient in children, dislocations and sprains are less common than in adults. d. Their bones have less blood flow.

C. Because soft tissues are resilient in children, dislocations and sprains are less common than in adults. A childs growth occurs because of an increase in size rather than an increase in the number of the muscle fibers. Fractures in children younger than 1 year are unusual because a large amount of force is necessary to fracture their bones. A childs bones have greater blood flow than an adults bones.

The nurse is working with a group of caregivers of school-age children discussing fractures. The nurse explains that if the fragments of fractured bone are separated, the fracture is said to be which of the following? a) Incomplete b) Spiral c) Greenstick d) Complete

Complete Correct Explanation: If the fragments of fractured bone are separated, the fracture is said to be complete. If fragments remain partially joined, the fracture is termed incomplete. Green stick fractures are one kind of incomplete fracture, caused by incomplete ossification, common in children. Spiral fractures twist around the bone.

A child is receiving morphine sulfate to control her postoperative pain after scoliosis repair. Which of the following findings is a side effect of morphine? A. Bradycardia B. Respiratory distress C. Severe hypotension D. Pruritis

D

During a 14-year-old adolescents physical examination, the nurse identifies that he plays soccer and football and is complaining of knee pain when he rises from a squatting position. The nurse should suspect: a. Legg-Calv-Perthes disease. b. osteomyelitis. c. Duchenne muscular dystrophy. d. Osgood-Schlatter disease.

D Knee pain and tenderness aggravated by activity that requires kneeling, running, climbing stairs, and rising from a squatting position are highly significant for Osgood-Schlatter disease. The etiology is believed to be related to repetitive stress from sports-related activities combined with overuse of immature muscles and tendons. Pain on activity that decreases with rest is indicative of Legg-Calv-Perthes disease. Preexisting pain, favoring the affected limb, erythema, and tenderness are associated with osteomyelitis. Duchenne muscular dystrophy presents with progressive generalized weakness and muscle wasting.

When performing physical assessments of children with musculoskeletal disorders, the nurse distinguishes normal variations in children's muscles versus adult muscles. These variations include: a) The infant's muscles account for 45% of total body weight as opposed to 25% of adult body weight. b) During adolescence, muscle growth is influenced by increased production of androgenic hormones. c) The young child has rigid soft tissue, so dislocations and sprains are common occurrences. d) Rapid bone and muscle growth in adolescents increase their agility, thereby decreasing the incidence of injuries.

During adolescence, muscle growth is influenced by increased production of androgenic hormones. Correct Explanation: During adolescence, muscle growth is influenced by hormonal changes, primarily the increased production of androgenic hormones. The infant's muscles account for only 25% of total body weight, whereas they account for 40% to 45% in an adult. The young child has resilient soft tissue, so dislocations and sprains are unusual occurrences. Rapid bone and muscle growth may contribute to the appearance of "clumsy" and awkward motions of the adolescent who is trying to adjust to new body dimensions.

Fracture of the femur typically occurs when a small child is lifted by one hand, as happens when a parent pulls on one arm to lift the child over a curb or up a step. a) False b) True

False Correct Explanation: If a small child is lifted by one hand, as happens when a parent pulls on one arm to lift the child over a curb or up a step, the head of the radius may escape the ligament surrounding it and become dislocated (nursemaid's elbow). Fracture of the femur is rare and is typically caused by an automobile accident, a fall from a considerable height, or child maltreatment.

A type of traction sometimes used in the treatment of the child with scoliosis is called which of the following? a) Dunlop's traction b) Russell traction c) Halo traction d) Bryant's traction

Halo traction Correct Explanation: When a child has a severe spinal curvature or cervical instability, a form of traction known as halo traction may be used to reduce spinal curves and straighten the spine. Halo traction is achieved by using stainless steel pins inserted into the skull while counter-traction is applied by using pins inserted into the femur. Weights are increased gradually to promote correction.

The nurse is caring for a child who has just had a plaster cast applied to the arm. The nurse is correct in doing which of the following with this child? a) Using only a draw sheet to move the casted arm. b) Handling the cast with open palms when moving the arm. c) Encouraging the child to move the arm slowly up and down to help the cast dry. d) Keeping a clove-hitch restraint gently tied on the hand to stabilize the arm.

Handling the cast with open palms when moving the arm. Correct Explanation: A wet plaster cast should be handled only with open palms because fingertips can cause indentations and result in pressure points. There is no reason the arm should be restrained or the arm moved to aid in the drying process.

A 2-year-old is diagnosed with osteomyelitis. Which of the following would you anticipate as a primary nursing intervention to include in the child's plan of care? a) Maintaining intravenous antibiotic therapy b) Assisting the child with crutch walking c) Keeping the child quiet while in skeletal traction d) Restricting fluid to encourage red cell production

Maintaining intravenous antibiotic therapy Correct Explanation: Osteomyelitis is a serious infection. It is treated vigorously with intravenous antibiotics. It would not require traction. The stem does not indicate the location of the infection, so the child may not need crutches. Fluid restriction does not help red blood cell production.

You meet a child with a slipped femoral epiphysis. In what type of child does this usually occur? a) Preadolescent girls b) Obese adolescent boys c) Tall, thin girls d) Active school-aged children

Obese adolescent boys Correct Explanation: A slipped epiphyseal femur injury most typically occurs in overweight preadolescent or adolescent boys. Stress increases the risk. A thin child would not have an increased risk, and the age range is past preadolescent and school age.

A nursing instructor is preparing a class presentation about tibia vara. Which of the following would the instructor include as a risk factor? a) Obesity b) Lack of sunlight exposure c) Late walking d) Hormonal alterations during puberty

Obesity Correct Explanation: Obesity is a risk factor for the development of tibia vara. Tibia vara occurs most frequently in children who are early walkers. Limited or lack of exposure to sunlight may lead to rickets. Hormonal alterations during puberty may play a role in the development of slipped capital femoral epiphysis.

A nurse is reviewing the medical record of a child who has sustained a fracture. Documentation reveals a bowing deformity. The nurse interprets this fracture as which of the following? a) Significant bending without actual breaking b) Incomplete fracture c) Bone buckling due to compression d) Bone that breaks into two pieces

Significant bending without actual breaking Correct Explanation: A plastic or bowing deformity is one in which there is significant bending of the bone without breaking. A buckle fracture is one in which the bone buckles rather than breaks. This is usually due to a compression injury. An incomplete fracture of the bone is a greenstick fracture. A complete fracture is one in which the bone breaks into two pieces

A 14-year-old girl with a fractured leg is receiving instructions from the nurse on how to use crutches. Which of the following interventions should the nurse implement to help prevent nerve palsy in the client? a) Be certain the child is walking with the crutches about 6 inches to the side of the foot b) Caution parents to clear articles such as throw rugs out of paths at home c) Teach the client not to rest with the crutch pad pressing on the axilla d) Assess the tips of the crutches to be certain the rubber tip is intact

Teach the client not to rest with the crutch pad pressing on the axilla Correct Explanation: Pressure of a crutch against the axilla could lead to compression and damage of the brachial nerve plexus crossing the axilla, resulting in permanent nerve palsy. Teach children not to rest with the crutch pad pressing on the axilla but always to support their weight at the hand grip. Always assess the tips of crutches to be certain the rubber tip is intact and not worn through as the tip prevents the crutch from slipping. Be certain the child is walking with the crutches placed about 6 inches to the side of the foot. This distance furnishes a wide, balanced base for support. Caution parents to clear articles such as throw rugs, small footstools or toys out of paths at home, to avoid tripping the child.

In caring for a child in traction, of the following interventions, which is the highest priority for the nurse? a) The nurse should record accurate intake and output. b) The nurse should provide age-appropriate activities for the child. c) The nurse should monitor for decreased circulation every four hours. d) The nurse should clean the pin sites at least once every eight hours.

The nurse should monitor for decreased circulation every four hours. Correct Explanation: Any child in traction must be carefully monitored to detect any signs of decreased circulation or neurovascular complications. Cleaning pin sites is appropriate for a child in skeletal traction. Providing age-appropriate activities and monitoring intake and output are important interventions for any ill child but would not be the highest priority interventions for the child in traction.

A nurse is conducting a physical examination on an 11-year-old boy with Legg-Calvé-Perthes disease. Which assessment finding would be expected? a) Loss of strength in ankle dorsiflexion b) Trendelenburg gait c) Lordosis d) Kyphosis

Trendelenburg gait Correct Explanation: The nurse would expect to note a Trendelenburg gait due to pain. Lordosis is an excessive curvature of the spine and is not associated with Legg-Calvé-Perthes disease. Kyphosis is an excessive curvature of the spine and is not associated with Legg-Calvé-Perthes disease. Loss of strength in ankle dorsiflexion is associated with some neuromuscular disorders but not this condition.

An infant is placed in Bryant's traction. For Bryant's traction to be effective, the infant must be positioned on the a) back with hips flat on the bed. b) stomach with both legs extended. c) back with the injured hip flexed and the uninjured one extended. d) back with hips up off the bed.

back with hips up off the bed. Explanation: For there to be traction, the infant's hips must be off the bed. On the stomach or hips on the bed are not the correct positions for this patient.

A girl with scoliosis is prescribed a body brace. The purpose of the brace is to a) improve spinal alignment. b) prevent herniation of a spinal disk. c) correct spinal curvature. d) prevent torticollis.

improve spinal alignment. Correct Explanation: Body bracing helps to hold the spine in alignment and prevent further curvature. The brace will not correct the problem. Herniation and torticollis are not associated with scoliosis.

While an adolescent wears a body brace for scoliosis, you would teach her a) that secondary sex changes will stop until the brace is removed. b) to continue with age-appropriate activities. c) to wear the brace a maximum of 20 hours each day. d) to stand absolutely still whenever she is out of the brace.

to continue with age-appropriate activities. Correct Explanation: Wearing a body brace should not interfere with normal activities, which are necessary to maintain adolescent self-esteem. Sex changes continue with or without bracing; the provider will determine the length of time for wearing the brace each day.

The nurse is reinforcing discharge teaching with the caregivers of a child who is going home after a cast has been applied. The nurse explains to the caregivers that which of the following should be reported if they occur or are seen related to this child? (Select all that apply) a) Drainage from under the cast b) Any pink color in the fingers or toes of casted extremity c) A foul odor under the cast d) Any area on the cast that is warm to the touch e) Any itching under or around the edges of the cast f) Looseness of the cast on the extremity

• Drainage from under the cast • A foul odor under the cast • Any area on the cast that is warm to the touch • Looseness of the cast on the extremity Correct Explanation: In addition to the five Ps, any foul odor or drainage on or under the cast, "hot spots" on the cast (areas warm to the touch), looseness or tightness, or any el evation of temperature must be noted, documented, and re ported. Family caregivers should be instructed to watch carefully for these same danger signals. Itching is common and does not need to be reported. Pink coloration of fingers and toes would be normal and not a concern.

The nurse is caring for a child after an accident in which the child fractured his arm. A cast has been applied to the child's right arm. Which of the following actions should the nurse implement? Select all that apply. a) Wear sterile gloves when removing or touching the cast. b) Monitor the color of the nail beds in the right hand. c) Check radial pulse in the both arms. d) Document any signs of pain. e) Wear a protective gown when moving the child's arm.

• Monitor the color of the nail beds in the right hand. • Check radial pulse in the both arms. • Document any signs of pain. Correct Explanation: Monitoring for signs of pain, decreased circulation, or change or variation in pulses in the extremity is important for the child in a cast. Pain can indicate serious complications, such as compartment syndrome. Wearing a gown or sterile gloves is unnecessary. Checking posterior pulses would be appropriate when a lower extremity is casted.

The nurse is doing patient teaching with a child who has been placed in a brace to treat scoliosis. Which of the following statements made by the child indicates an understanding of the treatment? a) "At least when I take a shower I have a few minutes out of this brace." b) "I am so glad I can take this brace off for the school dance." c) "When I start feeling tired, I can just take my brace off for a few minutes." d) Wearing this brace only during the night won't be so embarrassing."

"At least when I take a shower I have a few minutes out of this brace." Correct Explanation: The brace worn to treat scoliosis is worn day and night and should be removed only very briefly, such as for showering. The child needs to be taught that the brace must be worn at all times, during the day as well as the night.

The nurse is reinforcing teaching with the caregivers of a child who has been placed in an external fixation device for the treatment of an orthopedic condition. Which of the following statements made by the caregivers indicate an understanding of the external fixation device? a) "If we see any drainage around the pins when we are cleaning them, we won't be concerned." b) "He is very sensitive about the way the device looks, I am glad that his clothes will fully cover it so his friends won't tease him." c) "We will have to get some of the elastic bandages to place around the pins and pin sites." d) "It will be hard, but we know our child will be in this device for a long time."

"It will be hard, but we know our child will be in this device for a long time." Correct Explanation: External fixation devices are sometimes left in place for as long as 1 year. The pin sites are left open to the air and should be inspected and cleansed every 8 hours. The child and caregiver should be able to recognize the signs of infection at the pin sites. The appearance of the pins puncturing the skin and the unusual appearance of the device can be upsetting to the child.

The nurse is caring for an 8-year-old girl in traction. She has been in an acute care setting for two weeks and will require an additional 10 days in the hospital. She is showing signs of regression with thumb sucking and pleas for her tattered baby blanket. Which of the following would be the most helpful intervention? a) "Do you want a book to read?" b) "Let's ask your mom to bring your friends for a visit." c) "Would you like a coloring book?" d) "You are too big to suck your thumb."

"Let's ask your mom to bring your friends for a visit." Correct Explanation: After two weeks in traction, a child can become easily bored and regress in social and personal skills. A visit from friends arranged by the girl's mother or supervised by the child-life specialist would help her adapt to her immobilized state. Telling the girl she is too big to suck her thumb is unhelpful. Suggesting a book or coloring book would be unhelpful at this point, as she has likely grown tired of books and coloring after two weeks.

After teaching the parents of a 6-year-old child about caring for a sprained wrist, which statement by the parents indicates the need for additional teaching? a) "We'll apply a warm moist compress to the wrist for 20 minutes at a time." b) "We'll use an elastic wrap to keep the swelling down." c) "We'll be sure she keeps her wrist elevated above her heart."

"We'll apply a warm moist compress to the wrist for 20 minutes at a time." Correct Explanation: Care for a sprain includes rest, ice, compression, and elevation. Cold therapy, not heat, is used for 20 to 30 minutes at a time, then removed for 1 hour and repeated for the first 24 to 48 hours. Compression via an elastic bandage, elevating above heart level, and limiting activity are appropriate measures.

The nurse is caring for a 10-year-old boy who plays on two soccer teams. He practices four days a week and his team travels to tournaments once a month. He has been diagnosed with a stress fracture in one of his vertebrae. Which of the following instructions is most important to emphasize to the boy and his parents? a) "Ice will help reduce the inflammation." b) "NSAIDs can help with pain control and inflammation." c) "You will need to see a physical therapist for stretching and strengthening exercises." d) "You and your coaches need to understand that you cannot play soccer for at least six weeks."

"You and your coaches need to understand that you cannot play soccer for at least six weeks." Correct Explanation: A child with an overuse injury needs to avoid the causative activity for six to eight weeks. The other suggestions are also important, but the nurse must emphasize to the boy and his parents that they must tell the coaches "no soccer for six weeks." In some situations, it is helpful to supply a written directive from the nurse or physician to help the parent avoid undue pressure from coaches.

A nurse enters a patient's room who has a suspected diagnosis of spinal muscular atrophy (SMA). Which finding by the nurse is most concerning? 1) The child's chest is visibly sunken inward. 2) The child appears to have an S-shaped sign. 3) The child has absent tendon reflexes. 4) The child has trouble initiating spontaneous movement.

1) Pectus excavatum can occur when the child breathes using the diaphragm without intercostal muscle support (paradoxical breathing). This causes the chest to funnel and the xiphoid process retracts, which further restricts respiratory development. Scoliosis, absent or diminished tendon reflexes, and diminished ability to initiate spontaneous movement are all expected in this patient. Page 816.

The nurse is caring for a child diagnosed with pectus excavatum post-op day 1 from a steel bar placement. Which finding by the nurse is most concerning? 1) Breath sounds are absent on the right side 2) The child is crying and says his chest hurts 3) The child is lying supine. 4) The child's mother is asking the child to roll to his right side and finish his homework.

1) This indicates a pneumothorax may have occurred as a post-operative complication, and this needs to be addressed immediately. The child should not roll to EITHER side for 4 weeks after surgery. However, we can encourage aerobic activity. Pain is expected after surgery.

While caring for a patient with a myelomeningocele, which of the following actions, if made by the pediatric nurse, is incorrect? Select all that apply: 1) The nurse reports the presence of clear fluid from the lesion. 2) The nurse places the child in supine position to prevent skin breakdown. 3) The nurse encourages that light-weight blankets be used in place of heavy blankets or coverings. 4) The nurse places a piece of plastic wrap below the meningocele. 5) The nurse moistens the sac with a saline-soaked piece of gauze.

2, 3 A warmer or isolette should be used in place of blankets, which may place too much pressure on the sac. Supine positioning should be avoided in patients with myelomeningocele, as this places excessive pressure on the spinal cord sac. Prone positioning is preferred.

A nurse prepares to care for a patient diagnosed with athetoid, or dyskinetic, cerebral palsy. Which of the following does the nurse expect to see? Select all that apply: 1) Hypertonicity of affected extremities 2) Drooling 3) Worsening of symptoms when the child gets stressed 4) Worm-like writhing 5) Exaggerated deep tendon reflexes

2, 3, 4 Others are characteristic of spastic cerebral palsy. The infant may also appear limp or flaccid with the face, neck, and tongue possibly affected.

The pediatric nurse knows that which of the following medications are commonly used for patients with cerebral palsy? Select all that apply: 1) Docusate sodium 2) Diazepam 3) Dantrolene sodium 4) Baclofen 5) Atropine

2, 3, 4, 5 Can also use botulinum toxin, but this is administered by a nurse practitioner or physician.

The mother of a 3 month old infant calls the clinic and says "why can't my baby hold his head up? Is he developmentally delayed?" Which response by the nurse is best? 1) "Your child is unlikely to be developmentally delayed, but we would like you to come to the clinic for a well-baby check-up." 2) "Infants do not typically hold their heads up on their own before age 6 months, so you have nothing to worry about." 3) "You sound very concerned. This may indicate a developmental delay, but we would like you to come to the clinic for a well-baby check-up and testing." 4) "Your infant may not hold his head up for a couple more months. This is nothing to be concerned about."

4) This isn't an ideal answer, and could be worded more therapeutically, but out of the four options, this is the BEST answer. Infants lose head lag around 4-5 months.

In caring for a child with a compound fracture, what should the nurse carefully assess for? a. Infection b. Osteoarthritis c. Epiphyseal disruption d. Periosteum thickening

A Because the skin has been broken, the child is at risk for organisms to enter the wound. The incidence of osteoarthritis and the chance of epiphyseal disruption are not increased with compound fracture. Periosteum thickening is part of the healing process and is not a complication.

A nurse is teaching parents the difference between pediatric fractures and adult fractures. Which observation is true about pediatric fractures? a. They seldom are complete breaks. b. They are often compound fractures. c. They are often at the epiphyseal plate. d. They are often the result of decreased mobility of the bones.

A Pediatric fractures seldom are complete breaks. Rather, childrens bones tend to bend or buckle. Compound fractures are no more common than simple fractures in children. Epiphyseal plate fractures are no more common than any other type of fracture. Increased mobility of the bones prevents children from having complete fractures.

What is a realistic outcome for the child with osteogenesis imperfecta? a. The child will have a decreased number of fractures. b. The child will demonstrate normal growth patterns. c. The child will participate in contact sports. d. The child will have no fractures after infancy.

A The biochemical defect associated with osteogenesis imperfecta causes a defect in the synthesis of collagen. The abnormal collagen results in incomplete bone development, placing the child at high risk for fracturing bones. Receiving safety education and wearing protective apparatus can decrease the number of fractures. Because of incomplete bone formation, children with osteogenesis imperfecta do not have normal growth patterns. The high risk of fractures and the abnormal growth patterns do not allow for active participation in contact sports. Osteogenesis imperfecta is a life-long disease process.

A child with osteomyelitis asks the nurse, What is a sed rate? What is the best response for the nurse? a. It tells us how you are responding to the treatment. b. It tells us what type of antibiotic you need. c. It tells us whether we need to immobilize your extremity. d. It tells us how your nerves and muscles are doing.

A The erythrocyte sedimentation rate (ESR) indicates the presence of inflammation and infectious process and is one of the best indicators of the childs response to treatment. Although the ESR indirectly identifies whether an antibiotic is needed, the organism involved dictates the type of antibiotic and the length of treatment. The ESR does not direct whether the extremity will be immobilized and will not evaluate neuromuscular status.

The nurse caring for a child with Osgood-Schlatter disease should evaluate the childs: a. knowledge of activity restrictions. b. understanding of traction. c. acceptance of life-long limitations. d. knowledge of skin care.

A The major component of treatment for Osgood-Schlatter disease is activity restriction for 6 weeks or more. Traction is not used for Osgood-Schlatter disease. This is a self-limiting disorder, not a life-long disorder. The problem usually disappears once growth stops. Although activity is restricted, the degree of restriction should not result in skin care problems.

The major concern guiding treatment for the child with Legg-Calv-Perthes disease is to: a. avoid permanent deformity. b. minimize pain. c. maintain normal activities. d. encourage new hobbies.

A The major concern related to Legg-Calv-Perthes disease is to prevent an arthritic process resulting from the flattening of the femoral head of the femur when it protrudes outside the acetabulum. The pain associated with Legg-Calv-Perthes disease decreases with increased rest, making activity restriction an important factor for these children. The priority concern for treatment is to prevent deformity through decreased activity. Selected hobbies that do not require physical activity are encouraged.

Discharge planning for the child with juvenile arthritis includes the need for: a. routine ophthalmological examinations to assess for visual problems. b. a low-calorie diet to decrease or control weight in the less mobile child. c. avoiding the use of aspirin to decrease gastric irritation. d. immobilizing the painful joints, which is the result of the inflammatory process.

A The systemic effects of juvenile arthritis can result in visual problems, making routine eye examinations important. Children with juvenile arthritis do not have problems with increased weight and often are anorexic and in need of high-calorie diets. They are often treated with aspirin. Children with arthritis can immobilize their own joints. Range-of-motion exercises are important for maintaining joint flexibility and preventing restricted movement in the affected joints.

A group of students are reviewing information about bone healing in children. The students demonstrate understanding of this information when they identify which of the following? a) A child's bones heal more quickly than those of an adult. b) The process of breaking down and forming new bone is decreased in children compared with adults. c) Callus production is slower but greater in amount in children than in adults. d) A fracture closer to the growth plate heals much slower than one in the metaphysis.

A child's bones heal more quickly than those of an adult. Correct Explanation: Bone healing occurs in the same fashion as in the adult, but it occurs more quickly in children because of the rich nutrient supply to the periosteum. The closer a fracture is to the growth plate, the more quickly the fracture heals. The capacity for remodeling (the process of breaking down and forming new bone) is increased in children compared with adults. Children's bones produce callus more rapidly and in larger quantities than do adults' bones.

The nurse is teaching a group of peers regarding different types of fractures. Which of the following best describes an open fracture? a) A fracture in which there is a single break in the bone without penetration of the skin. b) A fracture in which the broken bone penetrates the skin. c) A fracture in which the fragments of the bone remain partially joined. d) A fracture in which the fragments of the bone are separated.

A fracture in which the broken bone penetrates the skin. Correct Explanation: The open fracture, also called a compound fracture, penetrates the skin. When the fragments of the bone are separated, the fracture is said to be complete.

A 3-year-old demonstrates lateral bowing of the tibia. Which of the following signs would indicate that the boy's condition is Blount disease rather than just the more typical developmental genu varum? a) A sharp, beaklike appearance to the medial aspect of the proximal tibia on x-ray b) The malleoli are touching c) The condition is bilateral d) The medial surfaces of the knees are more than 2 in apart

A sharp, beaklike appearance to the medial aspect of the proximal tibia on x-ray Correct Explanation: Blount disease is retardation of growth of the epiphyseal line on the medial side of the proximal tibia (inside of the knee) that results in bowed legs. Unlike the normal developmental aspect of genu varum, Blount disease is usually unilateral and is a serious disturbance in bone growth that requires treatment. In those with Blount disease, the medial aspect of the proximal tibia will show a sharp, beaklike appearance. The other answers all describe genu varum, not Blount disease.

Which of the following does the nurse understand places the child with myelomeningocele at high risk for infection? A) Neurogenic bladder B) Bowel incontinence C) Latex allergy D) Exposure of sac E) Corticosteroid use

Answer: A and D. Neurogenic bladder is the failure of the bladder to either store urine properly or empty itself of urine. Because of this urinary stasis occurs in the bladder placing the child at risk for infection. In myelomeningocele, the spinal cord is exposed, placing the child at high risk for infection. Immediate surgery is needed to help prevent infection from occurring. Bowel incontinence often occurs in children with myelomeningocele, but does not pose the same health risks as urinary incontinence. Latex allergy, although common, would not promote risk for infection alone. Corticosteroid use is not common in children with myelomeningocele.

The 7 year old patient has had a cast on to heal his fracture of his arm. After the expected time period, the nurse is teaching a child about what to expect when removing his cast. Which of the following teaching points should the nurse include? Select All that Apply: A) "The cast could begin to feel really warm as the striker saw is taking the cast off" B) "The striker saw will be very loud" C) "Look, see, the saw won't be able to cut your skin" D) "Once the cast is removed we will soak your leg in warm water" E) "you will still need to keep your leg very still even after the cast is removed"

Answer: A, B, C, and D. The cast/extremity under the cast could begin to feel warm during the cast removal process. The saw could be loud, but the nurse should demonstrate on him/herself that the saw can't won't cut the child. Once the cast is removed, it will be soaked and washed in warm soapy water, and it should be soaked in warm water daily. All of these need to be communicated to the child on the level of their understanding. The child should be told to start increasing activity the limb to regain strength and range of motion.

Which of the following could the nurse do to assess for hypotonia of the 4 month-old infant? SATA: A) Pick up the child and see if the child feels like it is slipping out of the nurse's grasp B) Assess to see if the child can momentarily support his own weight when placed in a standing position C) Hold the child up and ask them to walk forward for a few steps D) Move the infant from the supine position to the sitting position and see if the child can hold up his own neck E) Move the infants muscles and note any muscle spasms not associated with the muscle movement

Answer: A, B, and D. Hypotonia is often an indicator that the child has some form of neuromuscular disorder. The feeling that the child is slipping out of the examiners hands is often as sign of hypotonia. By four months the child is often able to support their own neck and to stand momentarily when their feet are placed on a flat surface and their body is in the vertical position. The child would not be able to walk on command at this age. Assessing for extraneous muscle spasms would indicate the presence of hypertonia, not hypotonia.

The nurse anticipates which of the following orders for the patient with Duchenne Muscular Dystrophy? Select all that apply: A) Prednisone B) Calcium supplements C) Bedrest D) Botulinum Toxin E) Chest percussion

Answer: A, B, and E. Corticosteroids are thought to help slow the progression of the disease. Calcium supplements are provided to help with the long-term effects of osteoporosis caused by the corticosteroids. Chest percussion can help remove excess secretions from the respiratory tract that the patient may be too weak to expel by themselves. Bedrest or use of botulinum toxin would not be recommended for this patient.

The nurse is caring for the patient with Russel's traction. Which of the following should the nurse include in this patients plan of care? SATA: A) Weight should remain off of the floor at all times B) Place a foot support to prevent foot drop C) Release traction for 5 minutes of every hour to provide skin care D) Ensure heel is resting on bed at all times E) Assess neurovascular status q 4 hours

Answer: A, B, and E. With all traction, the weights should remain off of the floor at all times and should not be released periodically or stopped for any reason unless emergent. A foot support will be needed for this patient because foot drop could develop related to the heel being elevated without support. The heel should be off of the bed at all times, not resting on it. Neurovascular status should be assessed often on this patient (as often as vitals are done).

Which of the following symptoms would the nurse expect to possibly see in the child with Duchenne muscular dystrophy? Select all that apply a) Protuberant belly b) Diminished intelligence c) Walking on the toes or balls of feet d) Gower's sign e) Spinal curvatures

Answer: A, C, D, and E. To protect balance, which is impaired in this disorder, the child may often have their belly sticking out with their shoulders pulled back. They often have a waddling gait and walk on the toes or on the balls of the feet. Gower's sign is the use of a special technique in order to rise off of the floor. Spinal curvatures often occur as the muscles in the body atrophy (including lordosis, kyphosis, and scoliosis). Intelligence is rarely affected by this disorder.

The nurse is providing discharge instructions to the family and the 5 year old child who has received a cast for his broken lower leg. Which statement, if made by the legal guardian would indicate the need for further teaching? A) I can place ice on the affected leg for the first 2 days for an hour at a time B) I should make sure that the leg is elevated C) We can use a blow dryer to help relieve the itching inside the cast D) We should call in if my kid complains of severe, unrelieved pain in his leg

Answer: A. Ice should only be used for periods of 20-30 minutes. All other pieces of information are correct (page 844).

You are the nurse working at a pediatrics clinic in Miami. You are assessing four amazingly awesome patients today. Which assessment finding concerns the nurse the most? A) The patient diagnosed with Duchenne Muscular Dystrophy demonstrating Gower's sign, waddling gait, and tachycardia B) The patient with cerebral palsy with scoliosis who is need of bracing C) The patient with myelomeningocele whose urine is cloudy and smells foul D) The patient with a brachial plexus injury who has an absent Moro's reflex

Answer: A. Tachycardia in the patient with Duchenne Muscular Dystrophy is a sign of heart muscle weakening, a serious complicating of this disorder. Scoliosis often occurs in patients with cerebral palsy. Cloudy, foul-smelling is a sign of a UTI, which is a complication of neurogenic bladder. This is concerning, but not as concerning as the heart trouble with Duchenne. An absent Moro's reflex would be expected in the patient with a brachial plexus injury.

The nurse is providing discharge instructions to the family and the 9 year old child who has received a Gore-tex cast for his broken lower leg. The mother states to the nurse, "how do I know if something is going wrong with the cast?" The nurse should teach the mother to call the physician if: A) The cast gets wet B) The toes become cold to the touch C) The child has a fever of 100 degrees for a couple hours D) The "petals" on the edge of the cast fall off E) The child feels extreme itchiness inside the cast

Answer: B, C, and E. Page 844. Because this is a Gore-tex cast, it is allowed to get wet and would not be a cause for concern. Any decrease in the neurovascular status of the limb should be reported including coolness of the toes. A temperature of 101.5 degrees or higher for 24 hours or longer should be reported. Because this is a gore-tex cast, no petals should be placed around the edges of the cast (page 843). Extreme itchiness within the cast would be reason to call.

A nurse is assessing a 3 year old child in the pediatrics wellness clinic, which of the following would be least concerning to the nurse? a) The child is holding his right arm close to his chest and refusing to use it b) When the child stands with his knees together, his ankles are far apart c) The child's pelvis drops slightly whenever he walks d) The child is in the shortest percentile for his age group

Answer: B. This condition, known as genu valgum or knock-knees, a common finding in children aged 2-3 and would not be concerning to the nurse. Refusal to use a limb could be a sign of damage or fracture to that limb. Trendelenberg's sign, or the drop of the pelvis when walking, could be a sign of hip dysplasia. Being in the shortest percentile for his age group could be a sign of a growth delay possibly caused by an underlying condition.

You, the awesome sauce nurse, are working at the new equally amazing pediatric clinic that opened literally five minutes from your house (nice commute!) your first patient is a child with marked pectus excavatum. What is the top nursing diagnosis for this patient? A) Impaired mobility B) Ineffective breathing pattern C) Disturbed body image D) Chronic pain

Answer: B. children with marked pectus excavatum should have a thorough assessment of their cardiac and pulmonary function, because the funnel chest can place pressure on both of these symptoms impairing proper expansion of the lungs. Page 849.

Little baby joe was born with an outer sac on his spine. Which of the following would correlate with a diagnosis of myelomeningocele? SATA: A) Accompanying hydrocephalus B) Leakage of the CSF C) Absence of deep tendon reflexes D) Constant dribbling of urine E) Meninges of the spine in the sac

Answer: C and D. Hydrocephalus could be present with either meningocele or myelomeningocele. Leakage of CSF would indicate a serious complication, not a common finding. The absence of deep tendon reflexes or the constant dribbling of urine indicates some neural involvement, differentiating it from meningocele. Meninges in the sac occurs in both myelomeningocele and meningocele. In myelomeningocele, the cord itself also protrudes into the sac

You are the nurse taking care of the infant just diagnosed with cerebral palsy. The mother of the child asks you, "What does this mean for my child?" What is the best response by the nurse? A) This means that you child will gradually lose more and more muscle mass until eventually they will be unable to sustain their respiratory function B) This is a disorder related to how your child was born. Most likely they sustained injury during the birthing process C) There really is no specific way to tell how this disease will affect your child other than it will affect the muscle tone and control in some way. D) Why are you asking me? I ain't no doctor!

Answer: C. Cerebral Palsy is a term used to describe a range of nonspecific clinical symptoms characterized by abnormal motor pattern and postures caused by nonprogressive abnormal brain function.

The nursing student is helping to take care of the infant with myelomeningocele. Which of the following actions, if made by the student nurse, should the nurse intervene? a) The nursing student uses and overhead radiant heater to warm the baby b) The student keeps the perineum clean and dry c) The student puts an absorbent diaper on the baby to keep it dry d) The nursing student turns props the baby slightly to the side with a towel under the abdomen

Answer: C. Diapering is not done for infants with myelomeningocele in order to prevent putting pressure on the sac. An overhead warmer should be used with this baby for temperature control. It is important to keep the perineum clean and dry. The child can be propped onto the side with a towel slightly to prevent skin breakdown from being in the prone position for long periods of time.

The nurse is assessing a young infant who comes in after being involved in a serious MVA. The nurse's priority would be? a) Assess for the presence of hypotonia or hypertonia of the neck b) Listen to respiratory sounds c) Brace the child's back to immobilize the child's spine d) Obtain a set of vital signs

Answer: C. It would be important to immobilize this child's spine.

The nurse is caring for the older child who has just undergone surgical correction of pectus excavatum. Which of the following order should the nurse anticipate doing all of the following except: a) Assessing and teaching correct use of PCA pump b) Assessing for symmetry of breath movement c) Keeping the patient on bedrest with bedside commode d) Teaching the child to avoid lying on her side

Answer: C. The child will be on strict bedrest for until the physician clears the child for activity. It is important that during this time the child keep from rolling on either side in order to keep the newly inserted bar in place. A PCA pump may be used in the older child to provide adequate pain relief. Assessing symmetry of breath movements would be important as pneumothorax is a complication of this surgery.

Tommy is a young child who is started walking early in life and usually is very active and happy. His mother tells you of a slow change that has happened to her son, and that he is less active than he has been. He now seems tired a lot and has difficulty doing things he used to do, such as running and playing. Which of the following would the nurse want to assess first? a) Check the child's back for dimpling or a tuft of hair at the base of the spine b) Assess the child's pain level and level of consciousness c) The child's ability to stand up and walk d) The presence of infantile reflexes

Answer: C. This child is presenting signs that most line up with a form of progressive muscular dystrophy, and it would be important for the nurse to follow up on the mother's claims that the child has difficulty ambulating and playing.

Which of the following actions of the pregnant woman would be most likely to affect the neuromuscular development of her unborn child in utero? A) The mother who says, "I drank a few alcoholic beverages in my second trimester, I just couldn't help it!" B) The 22 year old mother who started taking folic acid before she ever got pregnant C) The 27 year old mother who was involved in a car accident which caused her to go into labor D) The 19 year old who says, "I did drugs pretty bad, but I stopped as soon as I knew I was pregnant,"

Answer: D. Early in gestation, during 3-4 weeks of pregnancy, the neural tubes of the embryo begin to develop and differentiate. This is a critical time for the unborn child, and things like drug or alcohol use are most likely to cause developmental disorders during this time period.

When assessing children at the pediatric clinic, which of the following would the nurse want to investigate further? A) The two year old who gained control of his head and neck muscles before he was able to walk B) The infant who has very brisk deep tendon reflexes C) The young child who trips and falls while running around D) The infant who demonstrates hypertonia soon after birth.

Answer: D. Hypertonia of muscles or hypotonia (high or low muscle tone) would be considered abnormal findings in children and could be indicative of an underlying problem. Myelination proceeds in a cephalocaudal and proximodistal fashion, so the child would be expected to gain head control before being able to walk. Brisk deep tendon reflexes, while abnormal in older children, is normal in young infants. Tripping and falling could cause injury in the child, but would overall be considered a normal finding as children are still gaining control of their muscle function.

The nurse expects to see all of the following when assessing the newborn infant except: a) Flexible Metatarsus adductus b) Pes Planus c) Internal tibial torsion d) Polydactyly

Answer: D. Polydactyly, or the presence of an extra digit, would be considered abnormal in a newborn child. Flexible metatarsus adductus, also known as in-toeing, would be expected in the newborn infant, and would resolve as the infant matures (page 833). Internal tibial torsion, or the slight bowing of the legs, would also be expected r/t in-utero positioning. Pes Planus, or flat-footedness also occurs in infants and disappears as the child matures.

Which of the following problems is most often associated with myelomeningocele? A. Biliary atresia B. Hydrocephalus C. Craniosynostosis D. Tracheoesophageal fistula

Answer: b. Hydrocephalus is a frequently associated anomaly in 80% to 90% of children.

Which of the following clinical manifestations of developmental dysplasia of the hip would be seen in the newborn? A. Lordosis B. Ortolani sign C. Trendelenburg sign D. Telescoping of the affected limb

Answer: b. In the newborn period, the dysplasia usually appears as hip joint laxity. During the Ortolani test, the examiner places forward pressure and then backward pressure on the trochanter. If the femoral head is felt to slip, dysplasia may be present. This test is most reliable from birth to 2 to 3 months.

A newborn with congenital clubfoot is being treated with successive casts. The parents ask why so many casts are required. The nurse should explain that: A. casts are needed for the traction. B. each cast is good for only 6 weeks. C. surgical intervention will not be necessary. D. They allow for gradual stretching of tight structures.

Answer: d. Serial casting is begun shortly after birth and before discharge from nursery. Successive casts allow for gradual stretching of skin and tight structures on the medial side of the foot. Manipulation and casting of the leg are repeated frequently (every week) to accommodate the rapid growth of early infancy.

While assessing the newborn infant, the nurse notices a dimple with a tuft of hair at the bottom of the spine. What is the nurse's priority action? a) Place the child in the prone position to protect the area b) Soak a sterile dressing and place it over the area c) Notify the physician d) Educate the family on what this means for the child

Answer: d. it would be important to educate the family on the presence of spina bifida occulta, a neural tube defect that typically is benign and is often considered a normal defect. It would be important to differentiate this form of spina bifida from the highly stigmatized forms of spina bifida such as myelomingocele.

The nurse is caring for a 6-year-old boy with Russell traction applied to his left leg. Which intervention would be most appropriate to prevent complications? a) Adjust the weights as needed. b) Clean and massage his entire leg daily. c) Provide pin care as needed. d) Assess the popliteal region carefully for skin breakdown.

Assess the popliteal region carefully for skin breakdown. Correct Explanation: The nurse would assess the popliteal region carefully for skin breakdown from the sling. The nurse would adjust the weights only per physician orders. Cleaning and massaging the skin is unrelated to care of the child with Russell traction. Russell traction is a form of skin traction, so there is no pin care.

A nurse is performing a physical examination of a child with a suspected fracture. Which assessment technique would the nurse be least likely to use? a) Palpation b) Observation c) Auscultation d) Inspection

Auscultation Explanation: The physical examination specific to fractures includes inspection, observation, and palpation. Auscultation is not used.

A 4-year-old has just had a plaster cast applied to a fractured left arm. The nurse provides instructions to the parents regarding care for the cast. Which statement by the parent indicates a need for further instruction" A. "The cast may feel warm as it dries." B. I can use lotion or powder around the cast edges to relieve itching." C. "A small amount of white shoe polish can touch up a soiled white cast." D. "If the cast becomes wet, a blow dryer set on the cool setting may be used to dry it."

B

The parent of a child with osteomyelitis asks how the child acquired the illness. Which is the best response by the nurse? A. "Direct inoculation of the bone from stepping barefoot on a sharp stick." B. "An infection from a scratched mosquito bite carried the infection through the bloodstream to the bone." C. "The blood supply to the bone was disrupted because of the child's diabetes." D. "An infection of the upper respiratory tract."

B

Which is the nurse's best explanation to the parent of a toddler who asks what a greenstick fracture is? A. "It is a fracture in the growth plate of the bone." B. "It is a fracture that does not go all the way through the bone." C. "Because children's bones are not fully developed, any fracture in a young child is called a greenstick fracture." D. "It is a fracture in which a complete break occurs in the bone and small pieces of bone are broken off."

B

Parents bring their 2-week-old infant to a clinic for treatment after a diagnosis of clubfoot was made at birth. Which statement by the parent indicated the need for further teaching regarding this disorder? A. "Treatment needs to be started as soon as possible." B. "I need to bring my child back to the clinic in 1 month for a new cast." C. "I need to come to the clinic every week with my infant for the casting." D. "I realize my infant will require follow-up care until he reaches skeletal maturity."

B actually, the cast is changed every 1-2 weeks

Which intervention is part of the discharge plan for a child with osteomyelitis? a. Instructions for a low-calorie diet b. A referral to a home healthcare agency c. Instructions for a high-fat, low-protein diet d. Instructions for the parent to return the child to team sports immediately

B Because the child with osteomyelitis often requires intravenous antibiotics at home, a home healthcare referral is appropriate. The child with osteomyelitis is on a high-calorie, high-protein diet. The child with osteomyelitis may need time for the bone to heal before returning to full activities.

During painful episodes of juvenile arthritis, a plan of care should include which nursing intervention? a. A weight-control diet to decrease stress on the joints b. Proper positioning of the affected joints to prevent musculoskeletal complications c. Complete bed rest to decrease stress to joints d. High-resistance exercises to maintain muscular tone in the affected joints

B Proper positioning is important to support and protect affected joints. Isometric exercises and passive range-of-motion exercises will prevent contractures and deformities. Children in pain often are anorexic and need high-calorie foods. Children with juvenile arthritis need a combination of rest and exercise and need to avoid high-resistance exercises. They also benefit from low-resistance exercises such as swimming.

A child who has fractured his forearm is unable to extend his fingers. The nurse knows that this: a. is normal following this type of injury. b. may indicate compartmental syndrome. c. may indicate fat embolism. d. may indicate damage to the epiphyseal plate.

B Swelling causes pressure to rise within the immobilizing device leading to compartmental syndrome. Signs include severe pain, often unrelieved by analgesics, and neurovascular impairment. It is not uncommon in the forearm, so the inability to extend the fingers may indicate compartmental syndrome. It is not normal that the child is unable to extend his fingers; this indicates neurovascular compromise of some type. Paresthesia or numbness or loss of feeling can indicate a neurovascular compromise and can result in paralysis. Fat embolism causes respiratory distress with hypoxia and respiratory acidosis. Paresthesia is not related to damage to the epiphyseal plate.

The mother of a 3-year-old tells the nurse she is concerned about her child's bow-legged appearance. What is the best response from the nurse? A. "A referral to a pediatric orthopedic physician is indicated immediately." B. "Is your child having any pain or difficulty walking?" C. "Do not worry about it; this is normal." D. "I would be concerned if I were you."

B. "Is your child having any pain or difficulty walking?"

The nurse is presenting an in-service to a group of peers on the topic of traction. The nurse asks the group to give examples of types of skin traction. The following types were named by the nurses. Which of the following is an example of a type of skeletal traction? a) Balanced suspension traction b) Bryant's traction c) Russell traction d) Buck extension traction

Balanced suspension traction Correct Explanation: Skeletal traction exerts pull directly on skeletal structures by means of a pin, wire, tongs, or other device surgically inserted through a bone. Examples of skeletal traction are 90-degree traction and balanced suspension traction. Dunlop's traction, sometimes used for fractures of the humerus or the elbow, can be either skin or skeletal traction. It is skeletal traction if a pin is inserted into the bone to immobilize the extremity. Skin traction applies pull on tape, rubber, or a plastic material attached to the skin, which indirectly exerts pull on the musculoskeletal system. Examples of skin traction are Bryant's traction, Buck extension traction, and Russell traction.

A child who has undergone spinal fusion for scoliosis complains of abdominal discomfort and has periods of vomiting. On further assessment, the nurse notes abdominal distention. The priority action by the nurse at this time would be to: A. administer an antiemetic B. Increase the intravenous fluids. C. Notify the health care provider (HCP). D. Place the child in Sim's position.

C

A child with osteosarcoma is going to receive chemotherapy before surgery. Which statement by the parents indicates they understand the side effect of neutropenia? A. "My child will be more at risk for diarrhea." B. "My child's hair will fall out." C. "My child will be more at risk for infection." D. "My child will need to remain hydrated."

C

A nurse is caring for a 5-year-old with a fracture of the tibia involving the growth plate. When providing information to the parents, the nurse should indicate that: A. The child will never be able to play contact sports. B. The fracture usually heals within 6 weeks without further complications. C. This is a serious injury that could cause long-term growth issues. D. Fractures involving the growth plate require pain medication.

C

The nurse should tell the parents of a child with Duchenne muscular dystrophy that some of the progressive complications include: A. dry skin, hirsutism, protruding tongue, and mental retardation. B. anorexia, gingival hyperplasia, and dry skin and hair. C. contractures, obesity, and pulmonary infections. D. trembling, frequent loss of consciousness, and slurred speech.

C

A 4-year-old child with a long leg cast complains of fire in his cast. The nurse should: a. notify the physician on his next rounds. b. chart the complaint in the nurses notes. c. notify the physician immediately. d. report the complaint to the next nurse on duty.

C A burning sensation under the cast is an indication of tissue ischemia. It may be an early indication of serious neurovascular compromise, such as compartment syndrome, that requires immediate attention. The childs presenting symptom requires immediate attention. Notifying the physician on the next rounds is inappropriate. Charting the complaint in the nurses notes is an inappropriate action. Careful notation of symptoms is important, but the priority action is to contact the physician. Communication across shifts is important to the continuing assessment of the child; however, this symptom requires immediate evaluation, and the physician should be contacted.

Which factor is important to include in the teaching plan for parents of a child with Legg-Calv-Perthes disease? a. It is a chronic disease with long-term sequelae. b. It affects children in the toddler stage. c. There is a disturbance in the blood supply to the femoral epiphysis. d. It is caused by a virus.

C Legg-Calv-Perthes disease is a self-limiting disease that affects the blood supply to the femoral epiphysis. The most serious problem associated with it is the risk of permanent deformity. Legg-Calv-Perthes disease is not a chronic disease. The disease process usually lasts between 1 and 2 years and is a disorder of growth. It is seen in children between 2 and 12 years of age. Most cases occur between 4 and 9 years of age. The etiology is unknown.

A 6-year-old patient in skeletal traction for a femur fracture has pain and edema of the thigh and is febrile. The nurse should suspect which condition? a. Meningitis b. Crepitus c. Osteomyelitis d. Osteochondrosis

C The most serious complication of skeletal traction is osteomyelitis. Clinical manifestations include complaints of localized pain, swelling, warmth, tenderness, or unusual odor. An elevated temperature may accompany the symptoms. The symptoms of meningitis include headache, photophobia, fever, nausea, and vomiting. Crepitus is the sandy or gravelly feeling noted when a broken bone is palpated. Osteochondrosis is a disorder of the epiphyses involving an interruption of the blood supply.

The nurse is caring for a 15-year-old boy after left lower extremity amputation surgery after a diagnosis of osteosarcoma. The patient reports that his "left foot is in severe pain." What should the nurse do first? A. Remind him that he no longer has a left foot. B. Provide emotional support C. medicate for pain as ordered D. reposition for comfort

C. medicate for pain as ordered

In understanding the development of the musculoskeletal system, the nurse recognizes that which of the following is implanted in a gel-like substance during fetal life? a) Ligaments b) Cartilage c) Joints d) Tendons

Cartilage Correct Explanation: During fetal life, tissue called cartilage, which is a type of connective tissue consisting of cells implanted in a gel-like substance, gradually calcifies and becomes bone.

A child with a right femur fracture is placed in skin traction until surgery can be performed. During assessment of the child, the nurse notes that the dorsalis pedis is absent on the right foot. Which action should the nurse take? A. Administer an analgesic B. Release the skin traction C. Apply ice to the extremity D. Notify the health care provider (HCP)

D

The nurse is caring for a school-aged child with Duchenne muscular dystrophy. Which would be the most appropriate nursing diagnosis? A. Anticipatory grieving B. Anxiety reduction C. Increased pain D. Activity intolerance

D

The nurse is planning care for a child recently admitted for Guillain-Barré Syndrome has paralysis affecting the respiratory muscles and requires mechanical ventilation. When the parents ask about the paralysis, what is the best response by the nurse? A. "It must be difficult to accept the permanency of your child's paralysis." B. "Your child will first regain the use of their legs and then their arms." C. "In addition to the paralysis, your child will experience sensory loss." D. "The paralysis caused by this disease is temporary but the recovery can take up to 2 years."

D

The nurse teaching the parents of a child newly diagnosed with juvenile idiopathic arthritis (JIA). The nurse would evaluate the teaching as successful when the parent is states that, "The disorder is caused by...: A. ...the breakdown of osteoclasts in the joint space causing bone loss." B. ...loss of cartilage in the joints." C. ...a build-up of calcium crystals in joint spaces." D. ...an immune-stimulated inflammatory response."

D

Which nursing assessment is appropriate for determining neurovascular competency? a. Degree of motion and ability to position the extremity b. Length, diameter, and shape of the extremity c. Amount of swelling noted in the extremity and pain intensity d. Skin color, temperature, movement, sensation, and capillary refill of the extremity

D A neurovascular evaluation includes assessing skin color and temperature, ability to move the affected extremity, degree of sensation experienced, and speed of capillary refill in the extremity. The degree of motion in the affected extremity and the ability to position the extremity are incomplete assessments of neurovascular competency. The length, diameter, and shape of the extremity are not assessment criteria in a neurovascular evaluation. Although the amount of swelling is an important factor in assessing an extremity, it is not a criterion for a neurovascular assessment.

Which statement is most correct with regard to childhood musculoskeletal injuries? a. After the injury is iced, the swelling decreases, indicating the injury is not severe. b. The presence of localized tenderness indicates a more serious injury. c. The more swelling there is, the less severe the injury is. d. The less willing the child is to bear weight, the more serious the injury is.

D An inability to bear weight on the affected extremity is indicative of a more serious injury. With a fracture, general manifestations include pain or tenderness at the site, immobility or decreased range of motion, deformity of the extremity, edema, and inability to bear weight. A decrease in swelling after icing does not identify the degree of the injury. Localized tenderness along with limited joint mobility may indicate serious injury, but an inability to bear weight on the extremity is a more reliable sign. The degree of swelling does not indicate how serious the injury is.

Patient and parent education for the child who has a synthetic cast should include which information? a. Apply a heating pad to the cast if the child has swelling in the affected extremity. b. Wrap the outer surface of the cast with an Ace bandage. c. Split the cast if the child complains of numbness or pain. d. Cover the cast with plastic and waterproof tape to keep it dry while bathing or showering.

D Damp skin is more susceptible to breakdown. The cast should be kept clean and dry. To prevent swelling, elevate the extremity and apply bagged ice to the casted area. Wrapping the outer surface with an Ace bandage is not indicated. If the child complains of numbness or pain, he should return immediately to the clinic or emergency department for an evaluation of neurovascular status.

What is the most important nursing intervention to identify and minimize compartment syndrome? A. Apply BP cuff above the cast. B. Treat pain with minimum amount needed to control it. C. Elevate arm at least 30 minute/hr D. Perform frequent neurovascular checks

D. Perform frequent neurovascular checks

A 14-year-old girl has been diagnosed with scoliosis with a curve of 30 degrees. What medical intervention will treatment include for this patient? A. Transcutaneous electrical muscle stimulation (TENS) B. Only exercise to increase muscle tone and posture C. Surgery with insertion of a Harrington rod D. Use of a Milwaukee brace

D. Use of a Milwaukee brace

What terminology applies when there is an intentional omission of verbal or behavioral actions that are necessary for development of a healthy self-esteem, including social or emotional isolation of a child? A. Physical neglect B. Emotional abuse C. Physical abuse D. Emotional neglect

D. emotional neglect

The nurse is assessing a 10-year-old girl recently fitted with a cast on her wrist. Which assessment finding would alert the nurse to a possible infection? a) Diminished pulse b) Pallor of the fingers c) Drainage on the cast d) Delayed capillary refill

Drainage on the cast Correct Explanation: Drainage on the cast could indicate an infection. Pale fingers would suggest impaired circulation. Delayed capillary refill would suggest impaired circulation. Diminished pulse would suggest impaired circulation.

The nurse is observing a 3-year-old boy who is sitting and playing in the waiting area of his pediatrician's office. The nurse calls the boy and his mother back for the boy's appointment. The boy rolls onto his stomach and pushes himself to his knees. Then he presses his hands against his ankles, knees, and thighs, walking up the front of his body, to stand. Which condition should the nurse suspect in this client? a) Duchenne muscular dystrophy b) Congenital myotonic dystrophy c) Juvenile arthritis d) Facioscapulohumeral muscular dystrophy

Duchenne muscular dystrophy Correct Explanation: By age 3, children with Duchenne muscular dystrophy can rise from the floor only by rolling onto their stomachs and then pushing themselves to their knees. To stand, they press their hands against their ankles, knees, and thighs (they "walk up their front"); this is a Gower sign. Symptoms of facioscapulohumeral muscular dystrophy begin after the child is 10 years old, and the primary symptom is facial weakness. The child becomes unable to wrinkle the forehead and cannot whistle. Congenital myotonic dystrophy begins in utero and typically leads to death before age 1 year because of inability to sustain respiratory function. The symptoms of juvenile arthritis are primarily stiff and painful joints.

The nurse caring for a patient in a body cast knows that immobility can cause contractures, loss of muscle tone, or fixation of joints. Which of the following nursing interdisciplinary interventions are recommended to help prevent these adverse conditions? a) Check for a normal capillary refill of 3 to 5 seconds on a daily basis to ensure there in adequate arterial supply. b) Encourage active and passive range-of-motion activities to prevent ineffective tissue perfusion. c) Encourage child to stifle cough and take shallow breaths to prevent ineffective breathing patterns. d) Give the patient large, frequent meals with decreased fiber and increased protein and Vitamin C.

Encourage active and passive range-of-motion activities to prevent ineffective tissue perfusion. Correct Explanation: The nurse should turn the patient and encourage active and passive range-of-motion activities to prevent ineffective tissue perfusion. The patient should be instructed to cough and breathe deeply to prevent respiratory complications. Normal capillary refill is 1 to 3 seconds. The patient should be given small, frequent meals with increased fiber, protein, and vitamin C to prevent malnutrition.

The nurse is caring for an active 11-year-old presenting with tenderness in the shoulder. He is the pitcher for his baseball team and complains of shoulder pain with active internal rotation but is able to continue past the pain with full range of motion. Based on these reported symptoms, the nurse is aware that the disorder is most likely which of the following? a) Sever's disease b) Epiphysiolysis of the distal radius c) Osgood-Schlatter disease d) Epiphysiolysis of the proximal humerus

Epiphysiolysis of the proximal humerus Correct Explanation: Epiphysiolysis of the proximal humerus is an overuse disorder that occurs with rigorous upper extremity activity such as pitching and causes tenderness in the shoulder. Osgood-Schlatter disease causes knee pain and painful swelling or prominence of the anterior portion of the tibial tubercle. Sever disease causes pain over the posterior aspect of the calcaneus. Epiphysiolysis of the distal radius is an overuse disorder that causes wrist pain. It is common in gymnasts.

In understanding the function of the musculoskeletal system, the nurse recognizes that which of the following allows for movement of the body parts? a) Joints b) Tendons c) Ligaments d) Cartilage

Joints Correct Explanation: Bones are attached to each other by connecting links called joints, which allow for movement of the body parts. Skeletal muscles attach to the bones, with a moveable joint between them. Tendons and ligaments hold the muscles and bones together. Cartilage is a type of connective tissue consisting of cells implanted in a gel-like substance, which gradually calcifies and becomes bone.

The nurse is caring for a child with rickets. Which diagnostic test result would the nurse expect to find in the child's medical record? a) Low serum calcium levels b) X-ray confirmation of adequate bone shape c) Low alkaline phosphate levels d) High serum phosphate levels

Low serum calcium levels Correct Explanation: With rickets, serum calcium and phosphate levels are low and alkaline phosphate levels are elevated. Radiographs show changes in the shape and structure of the bone.

The nurse is caring for a child admitted with Legg-Calvé-Perthes disease. Which of the following clinical manifestations would likely have been noted in the child with this diagnosis? a) Pain in the groin and a limp b) Difficulty standing and walking c) Poor posture and malformed vertebrae d) Inflammation of the joints

Pain in the groin and a limp Correct Explanation: Symptoms first noticed in Legg-Calvé-Perthes disease are pain in the hip or groin and a limp accompanied by muscle spasms and limitation of motion.

The nurse is doing neurovascular checks on a child who has had a cast applied to treat a fracture. The nurse observes for diminished or absent sensation and numbness or tingling. In doing this the nurse is monitoring for which of the following symptoms? a) Pallor b) Paralysis c) Pain d) Paresthesia

Paresthesia Correct Explanation: Paresthesia is diminished or absent sensation or numbness or tingling. Pallor is paleness of color and paralysis is the loss of function.

The nurse is caring for a 10-year-old child in traction. After performing a skin assessment, she notices that the skin over the calcaneus appears slightly red and irritated. Which of the following should be the first intervention? a) Reposition the child's foot on a pressure-reducing device. b) Gently massage his foot and heel each shift. c) Apply lotion to his foot and avoid friction to the area. d) Make sure the skin and linens are clean and dry.

Reposition the child's foot on a pressure-reducing device. Correct Explanation: The nurse's first action is to remove continuous pressure from this area. The other actions can help decrease potential for skin breakdown, but the pressure must be relieved first.

The type of traction in which a pin, wire, tongs, or other device is surgically inserted through a bone is which of the following? a) Russell traction b) Skeletal traction c) Buck extension traction d) Skin traction

Skeletal traction Correct Explanation: Skeletal traction exerts pull directly on skeletal structures by means of a pin, wire, tongs, or other device surgically inserted through a bone. Skin traction applies pull on tape, rubber, or a plastic material attached to the skin, which indirectly exerts pull on the musculoskeletal system. Examples of skin traction are Bryant's traction, Buck extension traction, and Russell traction.

The nurse is discussing types of treatment used when working with children who have orthopedic disorders. Which of the following forms of treatment covers the lower part of the body, usually from the waist down, and either one or both legs while leaving the feet open? a) External fixation device b) Stockinette c) Internal fixation device d) Spica cast

Spica cast Correct Explanation: The hip spica cast covers the lower part of the body, usually from the waist down, and either one or both legs while leaving the feet open. The cast maintains the legs in a frog-like position. Usually, there is a bar placed between the legs to help support the cast.

The nurse is working with an 8-year-old girl who recently developed juvenile arthritis. The mother of the girl tells the nurse that she understands that exercise is important to help preserve muscle and joint function and asks the nurse for recommendations on types of exercise that would be appropriate. Which of the following should the nurse recommend? a) Jumping jacks b) Swimming c) Hiking d) Soccer

Swimming Correct Explanation: Swimming and tricycle or bicycle riding are excellent exercises because they provide smooth joint action. In contrast, to reduce joint destruction, activities that place excessive strain on joints, such as running, jumping, prolonged walking, and kicking, should be avoided.

A nurse is assessing a newborn and observes webbing of the fingers and toes. The nurse documents this finding as which of the following? a) Polydactyly b) Pectus carinatum c) Metatarsus adductus d) Syndactyly

Syndactyly Correct Explanation: Syndactyly refers to webbing of the fingers and toes. Polydactyly refers to the presence of extra digits on the hand or foot. Metatarsus adductus is a medial deviation of the forefoot. Pectus carinatum is a protuberance of the chest wall.

Healthcare providers are considered mandatory reporters in cases of identified or suspected child abuse. TRUE FALSE

TRUE

The nurse is caring for a child diagnosed with Legg-Calvé-Perthes disease. Of the following nursing interventions, which would be most important for the nurse to include in working with this child and the child's caregivers? a) The nurse should support the caregivers in restricting activity during the treatment. b) The nurse should help the caregivers to understand and the child to effectively use the corrective devices. c) The nurse should be a contact person when the child is hospitalized. d) The nurse should provide information when the child or caregiver requests it.

The nurse should help the caregivers to understand and the child to effectively use the corrective devices. Correct Explanation: Nursing care focuses on helping the child and caregivers to manage the corrective device and on the importance of compliance to promote healing and to avoid long-term disability.

A neonatal nurse examines an infant and notes decreased hip motion that causes pain upon movement. This nurse suspects Legg-Calvé-Perthes disease, a common pediatric hip disorder that causes pain and decreased hip motion, possibly leading to a femoral head deformity. a) False b) True

True Correct Explanation: Legg-Calvé-Perthes disease is a common pediatric hip disorder that causes pain and decreased hip motion, possibly leading to a femoral head deformity. It has an incidence of 1 per 1,200 live births, with some hereditary factors influencing incidence.

The nurse is caring for a 3-year-old boy with a fracture of the humerus. His chart indicates "fracture is partially through the physis extending into the metaphysis." The nurse identifies this as which Salter-Harris classification? a) Type I b) Type V c) Type II d) Type IV

Type II Correct Explanation: According to the Salter-Harris classification, a type II fracture is partially through the physis extending into the metaphysis. A type I fracture is through the physis, widening it. A type IV fracture is through the metaphysis, physis, and epiphysis. A type V fracture is a crushing injury to the physis.

A 14-year-old girl is diagnosed as having scoliosis. When doing scoliosis screening with her, an important observation would be to note a) her posterior spine when she bends forward. b) the angle of the iliac crest when she bends forward. c) the posterior spine when she bends sideways. d) the angle of her lower chest when she sits down.

her posterior spine when she bends forward. Correct Explanation: A lateral curvature of the spine (scoliosis) is best revealed when the child bends forward. Bending to the side would not provide an accurate assessment of the spine nor would assessing the iliac crest or the chest.

A nurse is working with a 12-year-old girl with osteomyelitis who is recovering from surgery. Which of the following are nursing interventions that should be implemented in this case? *(Select all that apply.) a) Instituting infection-control precautions related to drainage tubes b) Administration of IV antibiotics at the hospital c) Instruction to the parents regarding how to care for an antibiotic IV line at home d) Casting of the affected limb e) Instruction to the parents regarding the importance of the child maintaining bed rest f) Instruction to the parents regarding proper traction of the limb

• Administration of IV antibiotics at the hospital • Instruction to the parents regarding how to care for an antibiotic IV line at home • Instruction to the parents regarding the importance of the child maintaining bed rest • Instituting infection-control precautions related to drainage tubes Explanation: Osteomyelitis is infection of the bone. Medical therapy includes limitation of weight bearing on the affected part, bed rest, immobilization, and a short administration of an IV antibiotic such as oxacillin (Bactocill), as indicated by the blood culture. Intravenous therapy is usually initiated in the hospital and then continued at home for as long as 2 weeks. When the child is discharged from the hospital, be certain to review with parents measures to care for the antibiotic intravenous line if this will be continued at home. Keep in mind young children are active, even if they are on bed rest so need age appropriate activities so they maintain rest, not activity. If a child had surgery and drainage tubes are in place, institute infection-control precautions, because the drain evacuates infected material. Neither casting nor traction is required for osteomyelitis.

The school nurse cares for children with overuse injuries and refers them for treatment. Which statements accurately describe conservative interventions to prevent or care for these types of injuries? Select all that apply. a) Avoid the causative activity for 6 to 8 weeks. b) Avoid using NSAIDs for pain control. c) Immobilize the muscles that are involved. d) Have the coach monitor the treatment program for sports injuries. e) Apply ice to the injured area to reduce inflammation. f) Link the chain of communication in the disciplinary approach.

• Avoid the causative activity for 6 to 8 weeks. • Apply ice to the injured area to reduce inflammation. • Link the chain of communication in the disciplinary approach. Explanation: Conservative treatment methods for the child with an overuse injury include avoiding the causative activity for 6 to 8 weeks and applying ice to the injured area to reduce the inflammation and irritation. NSAIDs (ibuprofen) are used for inflammation and pain control. The physical therapist institutes a stretching and strengthening program for the appropriate muscle groups. Parents and coaches may not understand that the level of activity that causes overuse symptoms varies from child to child. Notes or telephone conversations from the physician or nurse to the child's coach can clarify any misconceptions about what is expected during the recovery and recuperative periods.

The nurse is caring for a 14-year-old boy in Buck traction for a slipped capital femoral epiphysis (SCFE). Which of the following would the nurse include when completing a neurovascular assessment of the affected leg? Select all that apply. a) Sensation b) Vital signs c) Color d) Capillary refill e) Pulse

• Color • Sensation • Pulse • Capillary refill Explanation: A neurovascular assessment includes assessing for color, movement, sensation, edema, and quality of pulses. Vital signs are not a component of a neurovascular assessment.

The nurse performing a focused health history on a newborn asks the parents if there are any hereditary disorders affecting musculoskeletal function in the family history. These disorders include (select all answers that apply): a) Clubfoot b) Scoliosis c) Hip dysplasia d) Limb deformities e) Brachial plexus injury

• Scoliosis • Clubfoot • Hip dysplasia Explanation: The nurse should explore the family history for any hereditary disorders. The presence of scoliosis, clubfoot, hip or skeletal dysplasia, or neuromuscular disorders in family members may help in diagnosing genetically linked orthopedic disorders.


Related study sets

Nursing 101 Chapter 42: Stress and Adaptation

View Set

Chapt. 14: Shock & Multiple Organ Dysfunction Syndrome

View Set

Amazon AWS Certified Solutions Architect - Associate SAA-C03 Exam Practice Questions

View Set

Criminal Justice Final (Chap. 6-8)

View Set

Chapter 16: Capacity and Legality

View Set

Strategic Management 492 chapter 4

View Set

Chapter #1 - Foundations of Economics

View Set

PSY test 3What is best defined as awareness of oneself and the environment?

View Set

LPI Linux Essentials 010 V1.6 - Assessment Test

View Set

Chapter 1 Exam - Basic Principles of Insurance

View Set

5.2 Explain importance of applicable regulations, standards or frameworks

View Set